Carlystern
Thanks Received: 1
Jackie Chiles
Jackie Chiles
 
Posts: 41
Joined: December 22nd, 2012
 
This post thanked 1 time.
 
 

Q12 - When a patient failed to respond

by Carlystern Thu Sep 26, 2013 5:43 pm

I chose (C), instead of the right answer (D)...

Still not sure why (C) is wrong and (D) is right....can someone break it down for me?
 
jlashley.ny
Thanks Received: 1
Vinny Gambini
Vinny Gambini
 
Posts: 1
Joined: March 04th, 2013
 
This post thanked 1 time.
 
 

Re: Q12 - When a patient failed to respond

by jlashley.ny Fri Sep 27, 2013 11:14 pm

Same problem -- I chose (C) and don't know why it's wrong!

The question asks how the doctor's initial hypothesis is supported by his "second set of recommendations and the results of its application".

Wasn't his "initial hypothesis" that the dosage needed to be increased? If so, then the herbal beverage was the only thing keeping his initial hypothesis from being correct! If the patient hadn't been drinking the beverage, the doctor's initial hypothesis would have had the intended effect. Isn't that exactly what (C) says?
User avatar
 
maryadkins
Thanks Received: 641
Atticus Finch
Atticus Finch
 
Posts: 1261
Joined: March 23rd, 2011
 
This post thanked 1 time.
 
 

Re: Q12 - When a patient failed to respond

by maryadkins Tue Oct 01, 2013 10:46 am

We were told that the doctor thought the dosage wasn't enough, so he doubled it to no avail (patient still sick!). Then the doctor gave a SECOND round of recommendations to be done simultaneously:

1. Lower the dosage back to the original amount.
2. Stop drinking the tea.

No change! Still sick.

Doctor gives THIRD round:

1. Double the dosage.
2. Don't drink the tea.

Bingo!

So how does this support the doctor's original hypothesis, which was that the dosage wasn't enough? Well, the dosage WASN'T enough. The beverage was a problem, too, or otherwise the patient would have responded to the doubling the first time around. But if the dosage had been enough, phase two (after the doctor's second round of recommendations) would have done the trick. (D) says this: the beverage wasn't the ONLY cause of the ineffectiveness of the medicine.

(C) doesn't support the doctor's original hypothesis: that the dosage wasn't sufficient. Remember that we're being asked to support that. We want evidence that the beverage was only partially responsible.

(A) is likewise wrong because we're not talking about the beverage.

(B) is inaccurate; the beverage did contribute.

(E) is also inaccurate and out of scope.
 
Djjustin818
Thanks Received: 0
Jackie Chiles
Jackie Chiles
 
Posts: 34
Joined: June 15th, 2012
 
 
 

Re: Q12 - When a patient failed to respond

by Djjustin818 Sun Oct 06, 2013 3:31 pm

But the stimulus is asking how the doctor's SECOND set of recommendations and it's result supported the doctor's initial hypothesis.

1st set: Double Dosage

2nd Set: initial dosage + stop drinking beverage

3rd Set: double dosage + stop drinking beverage

Because the results of the 2nd set was that there was no change, I chose B.. that the beverage contributing to the medicine's ineffectiveness was less plausible and it was something else. I think I may have gotten the sets mixed up some how but when I re-read the stimulus, I still see SECOND set. Am I missing something here?
User avatar
 
maryadkins
Thanks Received: 641
Atticus Finch
Atticus Finch
 
Posts: 1261
Joined: March 23rd, 2011
 
 
 

Re: Q12 - When a patient failed to respond

by maryadkins Tue Oct 08, 2013 5:10 pm

Yes, it's the second round of recommendations you're thinking of--but the results of those didn't show that the beverage WASN'T contributing to the effectiveness of the medicine. The patient stopped drinking the beverage but still didn't improve--so the beverage WAS having an effect.

Make more sense? That's why (B) doesn't work.
 
hychu3
Thanks Received: 3
Vinny Gambini
Vinny Gambini
 
Posts: 20
Joined: June 01st, 2013
 
 
 

Re: Q12 - When a patient failed to respond

by hychu3 Sat Oct 12, 2013 11:32 am

Hi,

What (C) says is true; the second set of recommendations does give evidence that the beverage was indeed responsible. This is why this answer is so attractive.

However, it's not as tailored as (D) in describing how the second set supports the doctor's hypothesis. The first set of recommendations seems to debunk the doctor's hypothesis. Why is it arguably true? Well, because the second set made it look like the dosage was not the only problem, as (D) says.

If you have a person who only knows about the first set of recommendations, and tell her just (C), then she would likely believe that the doctor's hypothesis is false; it's not a problem of dosage, but of the beverage. So, in that sense (C) is left in its "unrefined" form.
 
timsportschuetz
Thanks Received: 46
Elle Woods
Elle Woods
 
Posts: 95
Joined: June 30th, 2013
 
 
trophy
First Responder
 

Re: Q12 - When a patient failed to respond

by timsportschuetz Fri Dec 06, 2013 7:24 pm

@hychu3: I don't think this is the reason for eliminating (C). Notice how the question only talks about dosages - not about the effectiveness of the actual medicine being prescribed. We want an answer that specifically discusses "dosage"! We do not care of the effectiveness of the medicine since the question only talks about the single medicine being prescribed.
 
rachellewrx
Thanks Received: 1
Vinny Gambini
Vinny Gambini
 
Posts: 14
Joined: June 10th, 2015
 
 
 

Re: Q12 - When a patient failed to respond

by rachellewrx Sat Jun 20, 2015 9:38 am

I chose B during PT. I still chose B during review. However, after working on it for 15 minutes, I think I finally understood why D is correct. I'll share my process below.

We actually have four states: initial state plus three sets.

initial state : initial dosage + tea = ineffective
1st set: double dosage + tea = ineffective ( by comparing this with the initial state, we can see the tea could be a cause of ineffectiveness. Because if it was not, then double dosage should have made the medication effective. Not sure if dosage is a cause yet. )
2nd set: initial dosage + no tea = ineffective ( by comparing this with initial state, we can see that dosage could be a cause. because the medication is ineffective when the patient takes initial dosage, with or without tea.)
3rd set: double dosage + no tea = effective (by comparing this with 2nd set, we know that dosage is indeed a cause. then by comparing this set with 1st set, we know that tea is indeed a cause, too)

But the question only concerns about the 2nd set which gives us one more possible cause besides tea. So D is correct.

2nd set did not suggest that tea is less likely to be a cause. What makes it less likely to be a cause is if we had some thing like " double dosage + tea = effective". But we do not have such thing in the stimulus. Hence, B is incorrect. Note that we can not exclude tea as a cause just because dosage looks like a cause from 2nd set. It's possible that both causes work together to make the medication ineffective. And 3rd set confirmed that it is indeed what happened.

Finally, I think this question is not cool at all. It should be in Logic Game section.
 
asafezrati
Thanks Received: 6
Atticus Finch
Atticus Finch
 
Posts: 116
Joined: December 07th, 2014
 
 
 

Re: Q12 - When a patient failed to respond

by asafezrati Thu Aug 13, 2015 7:59 pm

The first hypothesis doesn't mention the beverage as a factor (the doctor doesn't even know about it at that point), and the second test+results by themselves don't support this either. You can understand the role of the beverage in the rest of the argument. This is why it's so confusing.

So do we need to consider the other information in the argument as well? It kinda messes up what the question stem asks for.
User avatar
 
maryadkins
Thanks Received: 641
Atticus Finch
Atticus Finch
 
Posts: 1261
Joined: March 23rd, 2011
 
 
 

Re: Q12 - When a patient failed to respond

by maryadkins Sat Aug 15, 2015 3:05 pm

I'm not sure what you're asking? You need all of the information in this argument to answer the question, yes. It's a series of steps, but they are all important/relevant.
 
kjsmit02
Thanks Received: 2
Vinny Gambini
Vinny Gambini
 
Posts: 16
Joined: January 07th, 2015
 
 
 

Re: Q12 - When a patient failed to respond

by kjsmit02 Mon Aug 17, 2015 7:55 pm

maryadkins Wrote:I'm not sure what you're asking? You need all of the information in this argument to answer the question, yes. It's a series of steps, but they are all important/relevant.

I think OP might be as confused as I am, in that while the question stem specifically tells us to address the doctor's second set of recommendations and its result, we are then further required to take this information and apply it to the argument as a whole. I originally chose answer (B). But when incorporating the entire argument and comparing it to the correct answer (D), I can see how (D) is the better choice when regarding the entire argument. I still feel that the second recommendation alone does exactly what (B) says: if you use the original dose without the beverage, it would give more support to the beverage not playing a role into the ineffectiveness of the medication, and thus supporting the initial hypothesis of the low dosage being the cause of the medications ineffectiveness. But when looking at the entirety of the argument, and at the final study, we see that it's really that both the tea and the dosage that were responsible for the ineffectiveness of the treatments, and that's addressed in (D). My issue is that why would the question stem specifically point out the second recommendation when asking about the argument as a whole? Wouldn't (D) be more of a description of how the 3rd recommendation supports the initial hypothesis/conclusion? Or maybe my logic is still off?
 
daijob
Thanks Received: 0
Elle Woods
Elle Woods
 
Posts: 74
Joined: June 02nd, 2015
 
 
 

Re: Q12 - When a patient failed to respond

by daijob Fri Aug 21, 2015 10:16 am

Djjustin818 Wrote:
1st set: Double Dosage

2nd Set: initial dosage + stop drinking beverage

3rd Set: double dosage + stop drinking beverage



So this is true right? And the question is asking about the 2nd set. From the set, I think I thought we might actually want to say "drinking beverage" may not be a contributing factor. And B says "less plausible"...I think we cannot say this, because it may or may not a contributing factor, we just cannot conclude anything. After seeing the 3rd set, we will see it is actually a contributing factor, but it was not only the contributing factor combining with the 1st set. So I guess...my question will be similar as others, I think we can conclude this only by combining all information of the stimulus, not only the 2nd set.
How can we know D is correct only by the second set??
User avatar
 
ohthatpatrick
Thanks Received: 3808
Atticus Finch
Atticus Finch
 
Posts: 4661
Joined: April 01st, 2011
 
This post thanked 1 time.
 
 

Re: Q12 - When a patient failed to respond

by ohthatpatrick Tue Sep 01, 2015 1:17 pm

VERY confusing stuff, I agree.

Initial hypothesis: the dose isn't high enough

1st move: let's up the dosage
result: still didn't work
conclusions: the dose STILL isn't high enough, or this person doesn't respond to this med for some reason

Suddenly, we find out the patient is drinking Herbalix.

"Oh, crap! Herbalix sometimes nullifies this med. Well, I'll definitely tell the patient to stop drinking Herbalix. But do I have the patient take a normal dose or a double dose? Well, I guess I should go back to the normal dose, because maybe the only reason the normal dose didn't work was because of the Herbalix."

2nd move: normal dose, no Herbalix
result: still didn't work
conclusions: "Okay, so it wasn't JUST that Herbalix was messing up the normal dosage."

"Herbalix may still have been messing up the double dose ... OR ... maybe this patient needs a triple dose (or more) ... OR ... maybe there's still something ELSE that is keeping the patient from responding to these meds."

Hopefully, you can see from this doctor's mock internal monologue, that after the 2nd move, it's still an open question whether or not Herbalix is having any effect on the medication's effectiveness. We won't discover until after the 3rd move that Herbalix WAS indeed having some effect on the effectiveness of the meds. After the 2nd move, we still are unsure about what effect it may have had.

What we DO know is that there's no way this patient was going to respond to the normal dose, with or without Herbalix.

So SOMETHING else must explain why the patient failed to respond to the prescribed original dose.

The 2nd move hasn't proven that "insufficient dosage" was the correct hypothesis. But the 2nd move proves that "Herbalix" is not the beginning and ending of the story.

-------
So you don't really need the 3rd move / observation to differentiate between (B) and (D).

(D) is a safer, more conservative judgment.

(B) would be fine if it said "They make it less plausible that the beverage alone contributed to the ineffectiveness of the original meds". Deciding at this point whether Herbalix "contributed at all" is premature. We'd have to do more testing, which is what the doctor then does with her 3rd move.

Hope this helps.
 
kyuya
Thanks Received: 25
Elle Woods
Elle Woods
 
Posts: 77
Joined: May 21st, 2015
 
 
 

Re: Q12 - When a patient failed to respond

by kyuya Sat Sep 05, 2015 7:02 pm

If you're like me this question was a nightmare during the timed sections. I'll try to break it down although it has been done above pretty well.

The question stem asks us why the second set of recommendation and the results of its application support the doctors initial hypothesis. So firstly, the way they ask this question is pretty convoluted in itself.

First you need to find the second set of recommendations, and also the doctors initial hypothesis.

1.) The second set of recommendations starts with the sentence "the doctor then advised to patient to resume"
2.) The doctors initial hypothesis was that the dosage was insufficient.

So how does #1 support #2?

If we look at the second set of recommendations, this is after the doctor realizes the patient is drinking a herbal beverage - a beverage he suspects is making the medicine not work. So, the second recommendation drops the dosage down to normal (remember, he initially doubled it before he realized the patient was drinking herbal) and then he orders the patient to stop drinking the tea.

The big question is ..why? Lets try to break this down. The doctor is attempting to rule out a cause and effect relationship - the one he suspects with the herbal tea cancelling out the initial dosage, making it not work. So when the patient stops drinking this herbal tea, and the medicine still doesnt work, it means that (1) either there is some other variable holding the medicine back from working, or (2) it wasn't the herbal tea that was making the medicine ineffective (at least at the initial dosage).

In other words, (D) actually states this, although not in a very out right manner. "...suggest that the beverage was not the only cause of the ineffectiveness of the prescribed dosage" . Isn't this exactly what (D) says? beverage was not the only reason it didn't work.. because even when we stopped drinking the beverage, the initial dosage wasn't enough.

BUT.. when we stopped the beverage and doubled the dosage, THEN the symptoms disappeared.
 
vickixu1994
Thanks Received: 0
Vinny Gambini
Vinny Gambini
 
Posts: 2
Joined: July 18th, 2016
 
 
 

Re: Q12 - When a patient failed to respond

by vickixu1994 Mon Oct 03, 2016 2:05 am

maryadkins Wrote:Yes, it's the second round of recommendations you're thinking of--but the results of those didn't show that the beverage WASN'T contributing to the effectiveness of the medicine. The patient stopped drinking the beverage but still didn't improve--so the beverage WAS having an effect.

Make more sense? That's why (B) doesn't work.


What proved that the beverage had an effect on the medicine is the comparison between [double dosage+ beverage] and [double dosage + no beverage]. one eliminated the symptoms the other did not. The difference between these two combinations is the presence/absence of beverage. This is why answer choice B doesn't work.

If the patient stopped drinking the beverage but still didn't improve, it actually casts doubt on the beverage's effect instead of confirming it. Think of it as a cause and effect relationship. If drinking and not drinking the beverage produces the same effect (no improvement), the effect is present both with and without the cause. how could this prove that the beverage was a cause?
 
andrewgong01
Thanks Received: 61
Atticus Finch
Atticus Finch
 
Posts: 289
Joined: October 31st, 2016
 
 
 

Re: Q12 - When a patient failed to respond

by andrewgong01 Fri May 05, 2017 2:19 am

I understand "D" but I do not understand why E has scope/accuracy issues.

The initial hypothesis was that the dosage was too low
. After removing the herbal beverage and at the lower dosage level nothing happened but at a higher dosage level it was successful . That does show that the beverage probably played some role intially, which is "D"

However, I think the fact that now the treatment does work after 1)Increasing the dosage and 2) Removing the beverage shows the doctor correctly ascertained the correct treatment, which is "E"


maryadkins Wrote:We were told that the doctor thought the dosage wasn't enough, so he doubled it to no avail (patient still sick!). Then the doctor gave a SECOND round of recommendations to be done simultaneously:

(E) is also inaccurate and out of scope.
User avatar
 
ohthatpatrick
Thanks Received: 3808
Atticus Finch
Atticus Finch
 
Posts: 4661
Joined: April 01st, 2011
 
This post thanked 1 time.
 
 

Re: Q12 - When a patient failed to respond

by ohthatpatrick Fri May 05, 2017 6:40 pm

The question stem wants us to support the doctor's initial hypothesis:

The dosage of X that I gave to Bob was insufficient.

Using the 2nd set of recommendations

Bob, take a normal dose of X, but stop drinking that herbal beverage.


The first time Bob had a normal dose of X, he showed no improvement (but he was drinking the herbal beverage).

The results of the application of the second set of recommendations are that Bob had a normal dose of X (not drinking the herbal beverage), and still showed no signs of improvement.

Again:

Initial: regular dose (w/ herbal bev), no improvement
1st rec: double dose (w/ herbal bev), no improvement
2nd rec: regular dose (w/ no herbal bev), no improvement

That's as much as we're allowed to use for these answer choices. (E) is saying that the last line there shows us that we're definitely using the right medication.

How does it show that? There has still been no improvement!

The doctor still has several paths to consider:
patient needs double dose (w/ no herbal bev)
patient needs more than double dose
patient needs different meds
 
MelisU651
Thanks Received: 0
Vinny Gambini
Vinny Gambini
 
Posts: 2
Joined: December 15th, 2019
 
 
 

Re: Q12 - When a patient failed to respond

by MelisU651 Sat May 09, 2020 8:31 am

ohthatpatrick Wrote:
That's as much as we're allowed to use for these answer choices. (E) is saying that the last line there shows us that we're definitely using the right medication.

How does it show that? There has still been no improvement!


I had chosen E for this question, and I see now that it might've been because I couldn't differentiate between the Dr.'s 2 and 3 recommendation.

But aside from that I feel like E is also irrelevant in that it doesn't have a direct connection to the Dr.'s recommendations or the results. The Dr. could have hypothesized anything that rules this possibility out, like let's decrease the dosage or eat more veggies etc. So this doesn’t necessarily describe how the Dr.’s initial hypothesis was correct.

I'm not confident that it's possible to eliminate E based on this so I would appreciate if you could help me clarify this.

P.S. Can also I use this opportunity to thank Patrick for his amazing explanations? Well here it goes: thanks a lot Patrick for providing us with such neat explanations, I always benefit a lot from them!